LSAT and Law School Admissions Forum

Get expert LSAT preparation and law school admissions advice from PowerScore Test Preparation.

User avatar
 Dave Killoran
PowerScore Staff
  • PowerScore Staff
  • Posts: 5852
  • Joined: Mar 25, 2011
|
#59786
Complete Question Explanation
(The complete setup for this game can be found here: lsat/viewtopic.php?t=26564)

The correct answer choice is (A)

This is a Local question, and thus you should reproduce a mini-setup next to the question. Since this is a Could Be True question, and we have two major templates produced by the MP split-block (one template with P delivered first, and another template with P delivered last), why not reproduce both templates, and show each with N delivered fourth, as follows:

pt26_s98_g2_q10.png
Note that the “#1” and “#2” designations are for our discussion purposes only; during the game you would not want to waste the time writing these designations out. An analysis of the two templates reveals that template #2 can never occur, and thus, that template is crossed out. If N is delivered fourth, then L would have to be delivered second (remember, we are only discussing template #2), but if L is delivered second, then the LO split-block dictates that O must be delivered fourth, and O cannot be delivered fourth in template #2 because N is already delivered there. Thus, in checking the answers, we should only refer to template #1.

In template #1 the only uncertainty involves L and O, and because this is a Could Be True question, you should immediately look at any answer that references L or O. Answer choice (A) references L, and since (A) could be true, it is the correct answer. Answer choice (B) also references L, but (B) cannot be true, and so it is incorrect. Answer choice (D) references O, but (D) cannot be true, and so it is also incorrect. Remember, once you make your mini-diagram in a Could Be True question, attack the answers by ignoring the variables that are placed. Instead, you can gain time by looking only at those answers that contain unplaced or moving variables (such as L and O in this problem).
You do not have the required permissions to view the files attached to this post.

Get the most out of your LSAT Prep Plus subscription.

Analyze and track your performance with our Testing and Analytics Package.